Compute $lim_{ntoinfty}frac{tfrac{n}{1}+tfrac{n-1}{2}+dots+tfrac{2}{n-1}+tfrac{1}{n}}{ln(n!)}$ [closed]












5












$begingroup$


How can I compute the following limit?
$$
lim_{ntoinfty}frac{dfrac{n}{1}+dfrac{n-1}{2}+dots+dfrac{2}{n-1}+dfrac{1}{n}}{ln(n!)}
$$



I have tried lots of methods, I can't get the answer.
Although I think the limit is $0$, I don't know how to explain it. Please, if someone could help me it would be fantastic.










share|cite|improve this question











$endgroup$



closed as off-topic by RRL, Holo, Namaste, Abcd, Zacky Dec 29 '18 at 12:41


This question appears to be off-topic. The users who voted to close gave this specific reason:


  • "This question is missing context or other details: Please provide additional context, which ideally explains why the question is relevant to you and our community. Some forms of context include: background and motivation, relevant definitions, source, possible strategies, your current progress, why the question is interesting or important, etc." – RRL, Holo, Namaste, Abcd, Zacky

If this question can be reworded to fit the rules in the help center, please edit the question.












  • 3




    $begingroup$
    Can you take a screenshot of what you tried and upload it to us? We like to see what your efforts have been so that we can better help you with this problem.
    $endgroup$
    – Noble Mushtak
    Dec 28 '18 at 21:04






  • 3




    $begingroup$
    For what it's worth, Wolfram Alpha says the sum on the top is $(n+1)H_n-n$. $H_napprox ln n$ for big n, so this gives us $nln n+ln n-n$ over $ln(n!)$. $ln(n!)$ is about $nln n$, so I'm guessing the limit is $1$. However, this is very rough reasoning, so don't trust me on that.
    $endgroup$
    – Noble Mushtak
    Dec 28 '18 at 21:07






  • 2




    $begingroup$
    OK, Wolfram Alpha confirms the limit is $1$.
    $endgroup$
    – Noble Mushtak
    Dec 28 '18 at 21:10










  • $begingroup$
    Hint: Use Stirling's approximation.
    $endgroup$
    – Breakfastisready
    Dec 28 '18 at 21:21










  • $begingroup$
    Thank you everybody!
    $endgroup$
    – M.Martinez
    Dec 29 '18 at 0:28
















5












$begingroup$


How can I compute the following limit?
$$
lim_{ntoinfty}frac{dfrac{n}{1}+dfrac{n-1}{2}+dots+dfrac{2}{n-1}+dfrac{1}{n}}{ln(n!)}
$$



I have tried lots of methods, I can't get the answer.
Although I think the limit is $0$, I don't know how to explain it. Please, if someone could help me it would be fantastic.










share|cite|improve this question











$endgroup$



closed as off-topic by RRL, Holo, Namaste, Abcd, Zacky Dec 29 '18 at 12:41


This question appears to be off-topic. The users who voted to close gave this specific reason:


  • "This question is missing context or other details: Please provide additional context, which ideally explains why the question is relevant to you and our community. Some forms of context include: background and motivation, relevant definitions, source, possible strategies, your current progress, why the question is interesting or important, etc." – RRL, Holo, Namaste, Abcd, Zacky

If this question can be reworded to fit the rules in the help center, please edit the question.












  • 3




    $begingroup$
    Can you take a screenshot of what you tried and upload it to us? We like to see what your efforts have been so that we can better help you with this problem.
    $endgroup$
    – Noble Mushtak
    Dec 28 '18 at 21:04






  • 3




    $begingroup$
    For what it's worth, Wolfram Alpha says the sum on the top is $(n+1)H_n-n$. $H_napprox ln n$ for big n, so this gives us $nln n+ln n-n$ over $ln(n!)$. $ln(n!)$ is about $nln n$, so I'm guessing the limit is $1$. However, this is very rough reasoning, so don't trust me on that.
    $endgroup$
    – Noble Mushtak
    Dec 28 '18 at 21:07






  • 2




    $begingroup$
    OK, Wolfram Alpha confirms the limit is $1$.
    $endgroup$
    – Noble Mushtak
    Dec 28 '18 at 21:10










  • $begingroup$
    Hint: Use Stirling's approximation.
    $endgroup$
    – Breakfastisready
    Dec 28 '18 at 21:21










  • $begingroup$
    Thank you everybody!
    $endgroup$
    – M.Martinez
    Dec 29 '18 at 0:28














5












5








5


1



$begingroup$


How can I compute the following limit?
$$
lim_{ntoinfty}frac{dfrac{n}{1}+dfrac{n-1}{2}+dots+dfrac{2}{n-1}+dfrac{1}{n}}{ln(n!)}
$$



I have tried lots of methods, I can't get the answer.
Although I think the limit is $0$, I don't know how to explain it. Please, if someone could help me it would be fantastic.










share|cite|improve this question











$endgroup$




How can I compute the following limit?
$$
lim_{ntoinfty}frac{dfrac{n}{1}+dfrac{n-1}{2}+dots+dfrac{2}{n-1}+dfrac{1}{n}}{ln(n!)}
$$



I have tried lots of methods, I can't get the answer.
Although I think the limit is $0$, I don't know how to explain it. Please, if someone could help me it would be fantastic.







limits






share|cite|improve this question















share|cite|improve this question













share|cite|improve this question




share|cite|improve this question








edited Dec 28 '18 at 22:15









Euler Pythagoras

54212




54212










asked Dec 28 '18 at 20:57









M.MartinezM.Martinez

343




343




closed as off-topic by RRL, Holo, Namaste, Abcd, Zacky Dec 29 '18 at 12:41


This question appears to be off-topic. The users who voted to close gave this specific reason:


  • "This question is missing context or other details: Please provide additional context, which ideally explains why the question is relevant to you and our community. Some forms of context include: background and motivation, relevant definitions, source, possible strategies, your current progress, why the question is interesting or important, etc." – RRL, Holo, Namaste, Abcd, Zacky

If this question can be reworded to fit the rules in the help center, please edit the question.







closed as off-topic by RRL, Holo, Namaste, Abcd, Zacky Dec 29 '18 at 12:41


This question appears to be off-topic. The users who voted to close gave this specific reason:


  • "This question is missing context or other details: Please provide additional context, which ideally explains why the question is relevant to you and our community. Some forms of context include: background and motivation, relevant definitions, source, possible strategies, your current progress, why the question is interesting or important, etc." – RRL, Holo, Namaste, Abcd, Zacky

If this question can be reworded to fit the rules in the help center, please edit the question.








  • 3




    $begingroup$
    Can you take a screenshot of what you tried and upload it to us? We like to see what your efforts have been so that we can better help you with this problem.
    $endgroup$
    – Noble Mushtak
    Dec 28 '18 at 21:04






  • 3




    $begingroup$
    For what it's worth, Wolfram Alpha says the sum on the top is $(n+1)H_n-n$. $H_napprox ln n$ for big n, so this gives us $nln n+ln n-n$ over $ln(n!)$. $ln(n!)$ is about $nln n$, so I'm guessing the limit is $1$. However, this is very rough reasoning, so don't trust me on that.
    $endgroup$
    – Noble Mushtak
    Dec 28 '18 at 21:07






  • 2




    $begingroup$
    OK, Wolfram Alpha confirms the limit is $1$.
    $endgroup$
    – Noble Mushtak
    Dec 28 '18 at 21:10










  • $begingroup$
    Hint: Use Stirling's approximation.
    $endgroup$
    – Breakfastisready
    Dec 28 '18 at 21:21










  • $begingroup$
    Thank you everybody!
    $endgroup$
    – M.Martinez
    Dec 29 '18 at 0:28














  • 3




    $begingroup$
    Can you take a screenshot of what you tried and upload it to us? We like to see what your efforts have been so that we can better help you with this problem.
    $endgroup$
    – Noble Mushtak
    Dec 28 '18 at 21:04






  • 3




    $begingroup$
    For what it's worth, Wolfram Alpha says the sum on the top is $(n+1)H_n-n$. $H_napprox ln n$ for big n, so this gives us $nln n+ln n-n$ over $ln(n!)$. $ln(n!)$ is about $nln n$, so I'm guessing the limit is $1$. However, this is very rough reasoning, so don't trust me on that.
    $endgroup$
    – Noble Mushtak
    Dec 28 '18 at 21:07






  • 2




    $begingroup$
    OK, Wolfram Alpha confirms the limit is $1$.
    $endgroup$
    – Noble Mushtak
    Dec 28 '18 at 21:10










  • $begingroup$
    Hint: Use Stirling's approximation.
    $endgroup$
    – Breakfastisready
    Dec 28 '18 at 21:21










  • $begingroup$
    Thank you everybody!
    $endgroup$
    – M.Martinez
    Dec 29 '18 at 0:28








3




3




$begingroup$
Can you take a screenshot of what you tried and upload it to us? We like to see what your efforts have been so that we can better help you with this problem.
$endgroup$
– Noble Mushtak
Dec 28 '18 at 21:04




$begingroup$
Can you take a screenshot of what you tried and upload it to us? We like to see what your efforts have been so that we can better help you with this problem.
$endgroup$
– Noble Mushtak
Dec 28 '18 at 21:04




3




3




$begingroup$
For what it's worth, Wolfram Alpha says the sum on the top is $(n+1)H_n-n$. $H_napprox ln n$ for big n, so this gives us $nln n+ln n-n$ over $ln(n!)$. $ln(n!)$ is about $nln n$, so I'm guessing the limit is $1$. However, this is very rough reasoning, so don't trust me on that.
$endgroup$
– Noble Mushtak
Dec 28 '18 at 21:07




$begingroup$
For what it's worth, Wolfram Alpha says the sum on the top is $(n+1)H_n-n$. $H_napprox ln n$ for big n, so this gives us $nln n+ln n-n$ over $ln(n!)$. $ln(n!)$ is about $nln n$, so I'm guessing the limit is $1$. However, this is very rough reasoning, so don't trust me on that.
$endgroup$
– Noble Mushtak
Dec 28 '18 at 21:07




2




2




$begingroup$
OK, Wolfram Alpha confirms the limit is $1$.
$endgroup$
– Noble Mushtak
Dec 28 '18 at 21:10




$begingroup$
OK, Wolfram Alpha confirms the limit is $1$.
$endgroup$
– Noble Mushtak
Dec 28 '18 at 21:10












$begingroup$
Hint: Use Stirling's approximation.
$endgroup$
– Breakfastisready
Dec 28 '18 at 21:21




$begingroup$
Hint: Use Stirling's approximation.
$endgroup$
– Breakfastisready
Dec 28 '18 at 21:21












$begingroup$
Thank you everybody!
$endgroup$
– M.Martinez
Dec 29 '18 at 0:28




$begingroup$
Thank you everybody!
$endgroup$
– M.Martinez
Dec 29 '18 at 0:28










2 Answers
2






active

oldest

votes


















5












$begingroup$

Let $H_n:= sumlimits_{k=1}^n frac{1}{k}$.



$ln(n!)sim n(ln(n)-1)$ by Stirling's formula.



As for the numerator, it is $nH_n-frac{1}{2}-frac{2}{3}-cdots-frac{n-1}{n}= nH_n-(n-1)+H_n-1$.



There are very strong estimations for $H_n$, for example $H_n= ln(n)+gamma+O(frac{1}{n})$.



Putting all this together yields that the limit is $1$, you can even obtain a nice error term that the sequence is in fact $1+O(frac{1}{ln(n)})$.






share|cite|improve this answer









$endgroup$













  • $begingroup$
    Thank you fo helping!!!
    $endgroup$
    – M.Martinez
    Dec 29 '18 at 0:28






  • 1




    $begingroup$
    I am glad I could help. If you think this answers your question, you should accept the answer.
    $endgroup$
    – A. Pongrácz
    Dec 29 '18 at 7:45



















3












$begingroup$

We can apply Stolz-Cesaro
taking $a_n=(n+1)H_n-n$ and $b_n=ln(n!)$:
$$l=lim_{nto infty} frac{a_{n+1}-a_n}{b_{n+1}-b_n}=lim_{ntoinfty} frac{(n+2)H_{n+1}-(n+1)-(n+1)H_n +n}{ln((n+1)!)-ln(n!)}=lim_{nto infty} frac{H_{n+1}}{ln(n+1)}$$
And since $H_n approx ln n+gamma +Oleft(frac1nright)$ we easily get:
$$l=lim_{nto infty} frac{a_{n+1}-a_n}{b_{n+1}-b_n}=1Rightarrow lim_{ntoinfty} frac{a_n}{b_n}=1$$






share|cite|improve this answer











$endgroup$









  • 1




    $begingroup$
    You have a few typos to fix, like $frac{(n+2)H_{n+1}...}{...}$
    $endgroup$
    – rtybase
    Dec 28 '18 at 22:03






  • 1




    $begingroup$
    One more, after all the simplifications, you should have $limlimits_{nrightarrowinfty}frac{H_{n+1}}{ln{(n+1)}}$
    $endgroup$
    – rtybase
    Dec 28 '18 at 22:11






  • 1




    $begingroup$
    Thank you for correcting me!
    $endgroup$
    – Zacky
    Dec 28 '18 at 22:14


















2 Answers
2






active

oldest

votes








2 Answers
2






active

oldest

votes









active

oldest

votes






active

oldest

votes









5












$begingroup$

Let $H_n:= sumlimits_{k=1}^n frac{1}{k}$.



$ln(n!)sim n(ln(n)-1)$ by Stirling's formula.



As for the numerator, it is $nH_n-frac{1}{2}-frac{2}{3}-cdots-frac{n-1}{n}= nH_n-(n-1)+H_n-1$.



There are very strong estimations for $H_n$, for example $H_n= ln(n)+gamma+O(frac{1}{n})$.



Putting all this together yields that the limit is $1$, you can even obtain a nice error term that the sequence is in fact $1+O(frac{1}{ln(n)})$.






share|cite|improve this answer









$endgroup$













  • $begingroup$
    Thank you fo helping!!!
    $endgroup$
    – M.Martinez
    Dec 29 '18 at 0:28






  • 1




    $begingroup$
    I am glad I could help. If you think this answers your question, you should accept the answer.
    $endgroup$
    – A. Pongrácz
    Dec 29 '18 at 7:45
















5












$begingroup$

Let $H_n:= sumlimits_{k=1}^n frac{1}{k}$.



$ln(n!)sim n(ln(n)-1)$ by Stirling's formula.



As for the numerator, it is $nH_n-frac{1}{2}-frac{2}{3}-cdots-frac{n-1}{n}= nH_n-(n-1)+H_n-1$.



There are very strong estimations for $H_n$, for example $H_n= ln(n)+gamma+O(frac{1}{n})$.



Putting all this together yields that the limit is $1$, you can even obtain a nice error term that the sequence is in fact $1+O(frac{1}{ln(n)})$.






share|cite|improve this answer









$endgroup$













  • $begingroup$
    Thank you fo helping!!!
    $endgroup$
    – M.Martinez
    Dec 29 '18 at 0:28






  • 1




    $begingroup$
    I am glad I could help. If you think this answers your question, you should accept the answer.
    $endgroup$
    – A. Pongrácz
    Dec 29 '18 at 7:45














5












5








5





$begingroup$

Let $H_n:= sumlimits_{k=1}^n frac{1}{k}$.



$ln(n!)sim n(ln(n)-1)$ by Stirling's formula.



As for the numerator, it is $nH_n-frac{1}{2}-frac{2}{3}-cdots-frac{n-1}{n}= nH_n-(n-1)+H_n-1$.



There are very strong estimations for $H_n$, for example $H_n= ln(n)+gamma+O(frac{1}{n})$.



Putting all this together yields that the limit is $1$, you can even obtain a nice error term that the sequence is in fact $1+O(frac{1}{ln(n)})$.






share|cite|improve this answer









$endgroup$



Let $H_n:= sumlimits_{k=1}^n frac{1}{k}$.



$ln(n!)sim n(ln(n)-1)$ by Stirling's formula.



As for the numerator, it is $nH_n-frac{1}{2}-frac{2}{3}-cdots-frac{n-1}{n}= nH_n-(n-1)+H_n-1$.



There are very strong estimations for $H_n$, for example $H_n= ln(n)+gamma+O(frac{1}{n})$.



Putting all this together yields that the limit is $1$, you can even obtain a nice error term that the sequence is in fact $1+O(frac{1}{ln(n)})$.







share|cite|improve this answer












share|cite|improve this answer



share|cite|improve this answer










answered Dec 28 '18 at 21:41









A. PongráczA. Pongrácz

6,0821929




6,0821929












  • $begingroup$
    Thank you fo helping!!!
    $endgroup$
    – M.Martinez
    Dec 29 '18 at 0:28






  • 1




    $begingroup$
    I am glad I could help. If you think this answers your question, you should accept the answer.
    $endgroup$
    – A. Pongrácz
    Dec 29 '18 at 7:45


















  • $begingroup$
    Thank you fo helping!!!
    $endgroup$
    – M.Martinez
    Dec 29 '18 at 0:28






  • 1




    $begingroup$
    I am glad I could help. If you think this answers your question, you should accept the answer.
    $endgroup$
    – A. Pongrácz
    Dec 29 '18 at 7:45
















$begingroup$
Thank you fo helping!!!
$endgroup$
– M.Martinez
Dec 29 '18 at 0:28




$begingroup$
Thank you fo helping!!!
$endgroup$
– M.Martinez
Dec 29 '18 at 0:28




1




1




$begingroup$
I am glad I could help. If you think this answers your question, you should accept the answer.
$endgroup$
– A. Pongrácz
Dec 29 '18 at 7:45




$begingroup$
I am glad I could help. If you think this answers your question, you should accept the answer.
$endgroup$
– A. Pongrácz
Dec 29 '18 at 7:45











3












$begingroup$

We can apply Stolz-Cesaro
taking $a_n=(n+1)H_n-n$ and $b_n=ln(n!)$:
$$l=lim_{nto infty} frac{a_{n+1}-a_n}{b_{n+1}-b_n}=lim_{ntoinfty} frac{(n+2)H_{n+1}-(n+1)-(n+1)H_n +n}{ln((n+1)!)-ln(n!)}=lim_{nto infty} frac{H_{n+1}}{ln(n+1)}$$
And since $H_n approx ln n+gamma +Oleft(frac1nright)$ we easily get:
$$l=lim_{nto infty} frac{a_{n+1}-a_n}{b_{n+1}-b_n}=1Rightarrow lim_{ntoinfty} frac{a_n}{b_n}=1$$






share|cite|improve this answer











$endgroup$









  • 1




    $begingroup$
    You have a few typos to fix, like $frac{(n+2)H_{n+1}...}{...}$
    $endgroup$
    – rtybase
    Dec 28 '18 at 22:03






  • 1




    $begingroup$
    One more, after all the simplifications, you should have $limlimits_{nrightarrowinfty}frac{H_{n+1}}{ln{(n+1)}}$
    $endgroup$
    – rtybase
    Dec 28 '18 at 22:11






  • 1




    $begingroup$
    Thank you for correcting me!
    $endgroup$
    – Zacky
    Dec 28 '18 at 22:14
















3












$begingroup$

We can apply Stolz-Cesaro
taking $a_n=(n+1)H_n-n$ and $b_n=ln(n!)$:
$$l=lim_{nto infty} frac{a_{n+1}-a_n}{b_{n+1}-b_n}=lim_{ntoinfty} frac{(n+2)H_{n+1}-(n+1)-(n+1)H_n +n}{ln((n+1)!)-ln(n!)}=lim_{nto infty} frac{H_{n+1}}{ln(n+1)}$$
And since $H_n approx ln n+gamma +Oleft(frac1nright)$ we easily get:
$$l=lim_{nto infty} frac{a_{n+1}-a_n}{b_{n+1}-b_n}=1Rightarrow lim_{ntoinfty} frac{a_n}{b_n}=1$$






share|cite|improve this answer











$endgroup$









  • 1




    $begingroup$
    You have a few typos to fix, like $frac{(n+2)H_{n+1}...}{...}$
    $endgroup$
    – rtybase
    Dec 28 '18 at 22:03






  • 1




    $begingroup$
    One more, after all the simplifications, you should have $limlimits_{nrightarrowinfty}frac{H_{n+1}}{ln{(n+1)}}$
    $endgroup$
    – rtybase
    Dec 28 '18 at 22:11






  • 1




    $begingroup$
    Thank you for correcting me!
    $endgroup$
    – Zacky
    Dec 28 '18 at 22:14














3












3








3





$begingroup$

We can apply Stolz-Cesaro
taking $a_n=(n+1)H_n-n$ and $b_n=ln(n!)$:
$$l=lim_{nto infty} frac{a_{n+1}-a_n}{b_{n+1}-b_n}=lim_{ntoinfty} frac{(n+2)H_{n+1}-(n+1)-(n+1)H_n +n}{ln((n+1)!)-ln(n!)}=lim_{nto infty} frac{H_{n+1}}{ln(n+1)}$$
And since $H_n approx ln n+gamma +Oleft(frac1nright)$ we easily get:
$$l=lim_{nto infty} frac{a_{n+1}-a_n}{b_{n+1}-b_n}=1Rightarrow lim_{ntoinfty} frac{a_n}{b_n}=1$$






share|cite|improve this answer











$endgroup$



We can apply Stolz-Cesaro
taking $a_n=(n+1)H_n-n$ and $b_n=ln(n!)$:
$$l=lim_{nto infty} frac{a_{n+1}-a_n}{b_{n+1}-b_n}=lim_{ntoinfty} frac{(n+2)H_{n+1}-(n+1)-(n+1)H_n +n}{ln((n+1)!)-ln(n!)}=lim_{nto infty} frac{H_{n+1}}{ln(n+1)}$$
And since $H_n approx ln n+gamma +Oleft(frac1nright)$ we easily get:
$$l=lim_{nto infty} frac{a_{n+1}-a_n}{b_{n+1}-b_n}=1Rightarrow lim_{ntoinfty} frac{a_n}{b_n}=1$$







share|cite|improve this answer














share|cite|improve this answer



share|cite|improve this answer








edited Dec 28 '18 at 22:13

























answered Dec 28 '18 at 21:56









ZackyZacky

7,87511062




7,87511062








  • 1




    $begingroup$
    You have a few typos to fix, like $frac{(n+2)H_{n+1}...}{...}$
    $endgroup$
    – rtybase
    Dec 28 '18 at 22:03






  • 1




    $begingroup$
    One more, after all the simplifications, you should have $limlimits_{nrightarrowinfty}frac{H_{n+1}}{ln{(n+1)}}$
    $endgroup$
    – rtybase
    Dec 28 '18 at 22:11






  • 1




    $begingroup$
    Thank you for correcting me!
    $endgroup$
    – Zacky
    Dec 28 '18 at 22:14














  • 1




    $begingroup$
    You have a few typos to fix, like $frac{(n+2)H_{n+1}...}{...}$
    $endgroup$
    – rtybase
    Dec 28 '18 at 22:03






  • 1




    $begingroup$
    One more, after all the simplifications, you should have $limlimits_{nrightarrowinfty}frac{H_{n+1}}{ln{(n+1)}}$
    $endgroup$
    – rtybase
    Dec 28 '18 at 22:11






  • 1




    $begingroup$
    Thank you for correcting me!
    $endgroup$
    – Zacky
    Dec 28 '18 at 22:14








1




1




$begingroup$
You have a few typos to fix, like $frac{(n+2)H_{n+1}...}{...}$
$endgroup$
– rtybase
Dec 28 '18 at 22:03




$begingroup$
You have a few typos to fix, like $frac{(n+2)H_{n+1}...}{...}$
$endgroup$
– rtybase
Dec 28 '18 at 22:03




1




1




$begingroup$
One more, after all the simplifications, you should have $limlimits_{nrightarrowinfty}frac{H_{n+1}}{ln{(n+1)}}$
$endgroup$
– rtybase
Dec 28 '18 at 22:11




$begingroup$
One more, after all the simplifications, you should have $limlimits_{nrightarrowinfty}frac{H_{n+1}}{ln{(n+1)}}$
$endgroup$
– rtybase
Dec 28 '18 at 22:11




1




1




$begingroup$
Thank you for correcting me!
$endgroup$
– Zacky
Dec 28 '18 at 22:14




$begingroup$
Thank you for correcting me!
$endgroup$
– Zacky
Dec 28 '18 at 22:14



Popular posts from this blog

Biblatex bibliography style without URLs when DOI exists (in Overleaf with Zotero bibliography)

ComboBox Display Member on multiple fields

Is it possible to collect Nectar points via Trainline?